LSAT and Law School Admissions Forum

Get expert LSAT preparation and law school admissions advice from PowerScore Test Preparation.

 Administrator
PowerScore Staff
  • PowerScore Staff
  • Posts: 8917
  • Joined: Feb 02, 2011
|
#38403
Complete Question Explanation
(The complete setup for this game can be found here: lsat/viewtopic.php?t=15027)

The correct answer choice is (D)

If T is 2, and S and T cannot be adjacent to each other, it follows that S must be one of the last three variables in our setup. The same holds true for V and W, which must occupy consecutive positions (5 and 6 or 6 and 7). The remaining variables (Q and R) must be 1 and 3, respectively, in compliance with the second rule:
PT76_O15 LG Explanations_Game #1_#3_diagram 1.png
The resulting two solutions prove that answer choice (D) could be true, and is therefore correct.
You do not have the required permissions to view the files attached to this post.

Get the most out of your LSAT Prep Plus subscription.

Analyze and track your performance with our Testing and Analytics Package.